Série dont la somme des carrés converge

Bonjour,

Est-il possible de trouver une série de réels positifs $\sum a_n$ telle que $\sum a_n^2$ converge et $\displaystyle \sum_{k=0}^N a_k \sim \sqrt N$?
«1

Réponses

  • Je pense que ton indice de sommation est $n$ au lieu de $k$.

    En étant un chouïa moins exigeant sur la somme partielle, celle-ci pourrait convenir (quitte à décaler, je ré-indice la somme pour la faire redémarrer à $2$) : on prend $a_n = (p \log p)^{-1/2}$ si $n=p$ est premier, $0$ sinon. Alors, on a vu il n'y a pas longtemps, dans un exercice que j'avais posé ici le mois dernier, que la série
    $$\sum_p a_p^2 = \sum_p \frac{1}{p \log p}$$
    converge, alors qu'avec le théorème des nombres premiers et un peu de calculs, on a, pour tout $x \geqslant 2$
    $$\sum_{p \leqslant x} a_p = \sum_{p \leqslant x} \frac{1}{\sqrt{p \log p}} = \frac{2 \sqrt x}{ (\log x)^{3/2}} + O \left( \frac{\sqrt x}{(\log x)^{5/2}} \right).$$
    Je rappelle qu'en arithmétique, la convention usuelle veut que tout produit ou toute somme indicée par la lettre $p$ ne porte que sur des nombres premiers.
  • Merci! On s’approche de $\sqrt N$... sans y être encore!
  • Bonsoir

    Je pense qu'une telle suite $(a_n)_n$ n'existe pas.
    Soit $(a_n) _{n \in \N}$ une suite de réels positifs telle que $ S_N: =\displaystyle \sum _{n = 0}^N a_n \underset{+\infty} {\sim} \sqrt N. \quad S_N = \sqrt N(1+ u _N) , \quad \lim_{N \to + \infty} u_N =0.$
    \begin{align*}
    a_{n +1}&= S_{n+1} - S_n = (\sqrt{n+1} - \sqrt n) (1+ u_n )+\sqrt{n+1}(u_{n+1} - u_n) .\\
    a_{n+1}^2 &= \left (\sqrt{n+1}- \sqrt n\right )^2 (1+ u_n)^2 + 2 \left(n+1-\sqrt{n^2+n} \right )(u_{n+1} - u_n)(1+u_n)+(n+1)(u_{n+1}-u_n)^2. \\
    a_{n+1}^2 &= \Big(\frac 1{4n}+o\big(\frac 1n \big) \Big) (1+u_n)^2 +(u_{n+1} - u_n)(1+u_n) + \Big( \frac 1{4n} + o\big (\frac1n \big)\Big)(u_{n+1} - u_n) (1+u_n) +(n+1) (u_{n+1} - u_n) ^2. \\
    a_{n+1}^2 &= \frac 1{4n} + o\big (\frac 1n \big) +( u_{n+1} - u_n) (1+u_n) + (n+1) (u_{n+1}-u_n)^2. \\ a_{n+1} ^2 &=\frac 1{4n} + o\big (\frac 1n \big) + (u_{n+1} - u_n) + \dfrac{u_{n+1}^2 -u_n^2}2 + (n+\frac 12)(u_{n+1} - u_n) ^2. \\ a_{n+1} ^2 &\geqslant \frac 1{4n} + o\big (\frac 1n \big) + (u_{n+1} - u_n) + \dfrac{u_{n+1}^2 -u_n^2}2.
    \end{align*}.
    Cette inégalité prouve que la série $\sum a_n^2$ diverge.
  • Lou16 Je ne comprends ta dernière conclusion :On ne peut rien dire sur la série de terme général $\frac 1{4n} + o\left (\frac
    1n \right)$ , non?
    Le 😄 Farceur


  • Bonsoir Gebrane,
    A partir d'un certain rang, $\dfrac 1 {4n} + o(\dfrac 1n) = \dfrac 1{4n}(1+ o(1))> \dfrac 1{8n} $ qui est le terme général d'une série divergente.
  • J'avais la "tété" ailleurs $\frac 1{4n} + o\left (\frac

    1n \right)\sim \frac 1{4n}$
    Le 😄 Farceur


  • Si $\sum_{k=0}^n a_k = \sqrt{n} + o(\sqrt n)$, alors d'après l'inégalité de Cauchy-Schwarz :
    $$
    \sum_{k=n+1}^{2n} a_k^2 \geqslant \frac{1}{n} \left(\sum_{k=n+1}^{2n} a_k\right)^2.
    $$
    Mais le membre de droite vaut $\frac1n\left(\sqrt{2n}-\sqrt{n} + o(\sqrt n)\right)^2 \xrightarrow[n\to\infty]{} (\sqrt 2 - 1)^2$.

    La série $\sum_{k\geq 0} a_k^2$ diverge donc.
  • Joli Siméon!

    J’ai essayé aussi d’utiliser l’inégalité de Cauchy-Schwarz mais je n’avais pas pensé à l’utiliser sur $S_{2n}-S_n$.
  • On peut aussi avoir des doutes grâce à la minoration évidente
    $$\sum_{k \leqslant x} a_k^2 \geqslant \left(\min_{k \leqslant x} a_k \right) \sum_{k \leqslant x} a_k \gg x^{1/2} \min_{k \leqslant x} a_k$$
    pour $x$ assez grand. C'est l'une des raisons pour laquelle j'ai cherché parmi mes sommes de nombres premiers, avec lesquelles on change (un peu) de monde.
  • Merci à tous pour ces preuves et aussi méthode pour « approcher l’équivalent ».
  • Je me demandais si on peut répondre entièrement à la question suivante généralisant la question initiale.

    On fixe $p\in\R_+$ et on considère une suite $(a_n)_{n\in\N}$ de réels positifs telle que $\displaystyle{\sum_{k=0}^n a_k \sim \sqrt{n}}$.
    Étudier la convergence de la série $\displaystyle{S_p=\sum_{n\geq 0} a_n^p}$.

    Nous avons démontré ci-dessus que la série $S_p$ diverge si $p\leq 2$. Peut-on dire quelque chose si $p>2$?
  • Dans ce post, vous avez montré (et je vous en remercie!), qu’il n’existe pas de série de réels positifs $\sum a_n$ telle que $\sum a_n^2$ converge et $\displaystyle \sum_{k=0}^N a_n \sim \sqrt N$.

    Est-il possible de prouver que pour une série de réels positifs telle que $\sum a_n^2$ converge, on a $\displaystyle \sum_{k=0}^N a_n=o(\sqrt N)$?.
  • Bonjour,

    Soit $\epsilon > 0$. On dispose d'un rang $n_0$ tel que $\sum_{n=n_0+1}^\infty a_n^2 < \epsilon$.

    Soit $N \geqslant n_0$. Alors d'après l'inégalité de Cauchy-Schwarz :
    $$
    \sum_{n=n_0+1}^N a_n \leqslant \sqrt{N-n_0} \sqrt{\sum_{n=n_0+1}^N a_n^2} \leqslant \sqrt N \sqrt \epsilon.
    $$
    De plus $\displaystyle \sum_{n=0}^N a_n = \sum_{n=0}^{n_0} a_n + \sum_{n=n_0+1}^N a_n$.

    On en déduit, que pour tout $N$ à partir d'un certain rang :
    $$
    0 \leqslant \frac1{\sqrt N}\sum_{n=0}^N a_n \leqslant 2\sqrt \epsilon
    $$
  • Merci Siméon!
  • Ce résultat implique le précédent sauf erreur?
  • Je suppose qu'il existe $0 < a < A$ tels que, pour tout $k \in \{1, \dotsc , n \}$, on ait $a \leqslant a_k \leqslant A$ (ce qui n'est pas une contrainte trop forte). Alors, d'après l'inégalité de Pólya-Szegö, il existe une constante $c_0 := c_0(p,a,A) > 0$ telle que, pour tout $n \geqslant 1$ assez grand
    $$\left( \sum_{k=1}^n a_k^p \right)^{1/p} \leqslant c_0 n^{-1+1/p} \sum_{k=1}^n a_k \leqslant c_1 n^{-1/2+1/p}$$
    de sorte que, si $n \geqslant 1$ est assez grand et si $p > 2$ est fixé
    $$\sum_{k=1}^n a_k^p \leqslant c_2 n^{1-\frac{p}{2}} \underset{n \to \infty}{\to} 0.$$
  • Merci noix de totos. Je ne connaissais pas cette inégalité.

    Du coup, si la série n’est pas nécessairement convergente pour $p>2$, il faut que je cherche du côté des séries lacunaires (comme souvent).
  • Oui.

    Quant à l'inégalité de Pólya-Szegö que j'utilise, c'est l'inégalité inverse de celle de Cauchy-Schwarz-Hölder qui, à mon sens, est tout aussi importante, mais moins connue.
  • Cher MrJ,

    Dans le cas $p > 2$ de ta question : http://www.les-mathematiques.net/phorum/read.php?4,1962114,1962468#msg-1962468, les deux situations sont possibles.

    Exemple 1. $\sum_{n \geq 1} \left(\frac{1}{2\sqrt n}\right)^p$ converge.
    Exemple 2. $\sum_{n\geq 1} (1_{n \text{ est un carré}})^p$ diverge.

    Et si on suppose la suite $a$ décroissante ? (indication en blanc ci-dessous)
    Montrer que dans ce cas on a nécessairement $a_n \sim \frac1{2\sqrt n}$.
  • @ndt : j'avoue que je ne connaissais pas cette inégalité de Pólya-Szegö, tu pourrais nous en dire un peu plus, ça a l'air très intéressant.
  • Oui.

    Inégalité de Pólya-Szegö. Soit $(a_k)$ et $(b_k)$ des suites de réels strictement positifs telles qu'il existe $0<a<A$ et $0<b<B$ tels que, pour tout $k$, on ait $a \leqslant a_k \leqslant A$ et $b \leqslant b_k \leqslant B$. Alors, pour tout $n \geqslant 1$
    $$\left( \sum_{k=1}^n a_k^2 \right)^{1/2} \left( \sum_{k=1}^n b_k^2 \right)^{1/2} \leqslant \frac{1}{2} \left( \sqrt{\frac{AB}{ab}} + \sqrt{\frac{ab}{AB}} \right) \sum_{k=1}^n a_kb_k.$$
    Admirez la beauté de cette inégalité.

    Inégalité de Pólya-Szegö généralisée. On reprend les suites $(a_k)$ $(b_k)$ ci-dessus, avec les mêmes hypothèses, et soit $\lambda > 1$. On note
    $$D_1 := \lambda \left( bAB^{\lambda/(\lambda-1)} - a B b^{\lambda/(\lambda-1)} \right) \quad \textrm{et} \quad D_2 := \frac{\lambda}{\lambda-1} \left( aBA^\lambda - bAa^\lambda \right)$$
    puis
    $$C_\lambda := \frac{A^\lambda B^{\lambda/(\lambda-1)} - a^\lambda b^{\lambda/(\lambda-1)}}{D_1^{1/\lambda} D_2^{1-1/\lambda}}.$$
    Alors
    $$\left( \sum_{k=1}^n a_k^\lambda \right)^{1/\lambda} \left( \sum_{k=1}^n b_k^{\lambda/(\lambda-1)} \right)^{1 - 1 / \lambda} \leqslant C_\lambda \sum_{k=1}^n a_kb_k.$$
    Je peux donner des références pour ceux qui le souhaitent. Il existe aussi des versions continues de l'inégalité de Pólya-Szegö.
  • Siméon

    Bonjour, voici un raisonnement simple donc surement faux - je n' utilises pas la décroissante de la suite
    Soit $S_n=\displaystyle \sum_{k=0}^n a_k \sim \sqrt n$ donc $S_n=\sqrt n.w_n$ avec $(w_n)$ une suite tendant vers 1.. donc $\displaystyle a_n=S_n-S_{n-1}=\sqrt n.w_n-\sqrt{n-1}.w_{n-1}\\
    =(\sqrt n-\sqrt{n-1})w_n+\sqrt{n-1}(w_n-w_{n-1})\\
    =(\sqrt n-\sqrt{n-1})w_n \big[1+\frac{\sqrt{n-1}}{(\sqrt n-\sqrt{n-1})}\frac{w_n-w_{n-1}}{w_n}\big]\sim \sqrt n-\sqrt{n-1}\\
    \sim \frac 12\frac 1 {\sqrt n}$

    J'ai la tête lourde pendant ce confinement, où est mon erreur ?
    Le 😄 Farceur


  • Tu n'utilises pas la décroissance, mais à l'avant-dernière étape tu sembles considérer que $n(w_{n}-w_{n-1}) \to 0$.
  • Merci , je vois ma boulette
    Le 😄 Farceur


  • Simeon d’après ton dernier message, on sent que la décroissance de la suite entraîne $n(w_{n}-w_{n-1}) \to 0$
    Dans ton raisonnement utilises-tu ce petit lemme qu'on applique à $u_n= w_{n}-w_{n-1}$
    Lemme a écrit:
    Si $(u_n)$ est une suite décroissante de nombres strictement positifs et si $\sum u_n $est convergente, alors lim n $u_n$ = 0
    Le 😄 Farceur


  • @ndt : Merci (tu).
  • @Siméon : Merci pour ta réponse! On ne peut rien dire de plus du coup.

    @noix de totos : Merci pour l'inégalité. J'avais essayé de la trouver sur internet, mais je tombais sur des inégalités en rapport avec les distributions (dont l'inégalité que tu nous as présenté ci-dessus ne semblait pas découler).
  • Voici des références.

    (i) Pour la $1$ère, évidemment l'incontournable

    Hardy, Littlewood, Pólya, Inequalities, Cambridge University Press, Cambridge 1934 (ou une édition plus récente), p. 143.

    (ii) Pour la $2$nde

    C. L.Wang, On development of inverses of the Cauchy and Hölder inequalities, SIAM Review 21 (1979), 550--557.
  • En réponse à : http://www.les-mathematiques.net/phorum/read.php?4,1962114,1962880#msg-1962880

    Cher gebrane,

    Il me semblerait que je t'ai égaré par mégarde. D'après ton calcul et mon indication, la décroissance de $u$ tel que $\sum_{k=0}^n u_k \sim \sqrt n$ implique bien que $n(w_n - w_{n-1}) \to 0$. Mais je ne sais pas du tout si c'est une piste intéressante... En revanche, s'inspirer de la démonstration du lemme que tu mentionnes est une bonne idée (je ne crois pas qu'on puisse l'appliquer directement) !
  • Merci Cher Simeon
    Le 😄 Farceur


  • Je crois avoir réussi avec la définition directement (EDIT : Finalement non...)

    Supposons que $(a_n)_{n\in\N}$ est une suite décroissante de réels positifs et que $S_n\sim \sqrt{n}$.
    Soit $(u_n)_{n\in\N}$ la suite définie par $a_n=u_n (2\sqrt{n})^{-1}$.
    Si $(u_n)_{n\in\N}$ ne converge pas vers $1$, alors il existe un réel $\varepsilon>0$ et une suite extraite strictement croissante $(k_n)_{n\in\N}$ telle que \[
    \Big(\forall n\in\N,\quad u_{k_n}>1+\varepsilon\Big)\quad\text{ou}\quad\Big(\forall n\in\N,\quad u_{k_n}<1-\varepsilon\Big).
    \] Pour tout $n\in\N$, on a dans le premier cas que \[
    S_{k_n} = S_{k_0} + \sum_{i=1}^n (S_{k_i}-S_{k_{i-1}})\geq S_{k_0} + \sum_{i=1}^n (k_i-k_{i-1}) a_{k_i}\geq S_{k_0}+(1+\varepsilon) \sum_{i=1}^n (k_i-k_{i-1})\dfrac{1}{2\sqrt{k_i}}.

    \] EDIT : Finalement, je me rend compte que je suis bloqué. Je voulais minorer avec une intégrale, mais ça ne marche pas...
  • @Siméon : Aurais-tu une indication? J'ai toujours une approximation trop faible pour en déduire l'équivalent dans chaque méthode que j'ai essayé. Merci!
  • Oui bien sûr : pour $\varepsilon > 0$, comparer $a_n$ avec
    $$
    \frac1{\lfloor \varepsilon n\rfloor} \sum_{k = n}^{n + \lfloor \varepsilon n\rfloor} a_k.
    $$
  • C’est plutôt efficace! Je n’avais pas pensé à utiliser un réel $\varepsilon$ de cette manière. Merci!

    Je me demande si on ne pourrait pas généraliser : si $(u_n)$ et $(v_n)$ sont deux suites positives et de même monotonie et que leur suite des sommes partielles sont équivalentes et divergentes, alors $(u_n)$ et $(v_n)$ sont équivalentes.

    Cela ferait une réciproque partielle à un résultat classique. Je réfléchirai demain s’il est possible de montrer ce résultat ou de construire un contre-exemple.

    Merci à nouveau pour ton aide Siméon.
  • bonsoir MrJ
    Peux-tu expliquer comment tu as utilisé l'indication de Simeon
    Le 😄 Farceur


  • Cher MrJ, la preuve se généralise facilement à tout $\alpha \in \mathbb R_+$ : pour toute suite positive monotone $u$,
    $$
    \frac1{n^\alpha}\sum_{k=0}^n u_k \xrightarrow[n\to\infty]{} 1 \implies \frac{nu_n}{n^{\alpha}} \xrightarrow[n\to\infty]{} \alpha.
    $$
    Ceci fonctionne encore en remplaçant $n \mapsto n^\alpha$ par $n \mapsto n^{\alpha} L(n)$ où $L$ est une fonction à variation lente. Mais on ne peut pas remplacer $n \mapsto n^\alpha$ par n'importe quelle fonction, même en la supposant convexe ou concave.

    Cher gebrane, je laisse MrJ te répondre.
  • Cher Simeon j'ai suivis ton indication mais je bloque sur un passage à la limite, j'ai besoin de savoir que la suite $\sqrt n u_n$ admet une limite
    Le 😄 Farceur


  • Cher gebrane, tu devrais normalement obtenir la minoration suivante en suivant l'indication : quel que soit $\varepsilon > 0$, lorsque $n \to \infty$,
    $$
    a_n \geqslant \frac{\sqrt{1+\varepsilon}-1}{\varepsilon \sqrt n}(1+o(1))
    $$

    Edit : leqslant/geqslant
  • J'obtiens en supposant que la dite limite existe que $$
    \forall \epsilon \in ]0,1[, \quad \frac{\sqrt{1+\varepsilon}-1}{\varepsilon} \leq \lim \sqrt n a_n \leq \frac{1-\sqrt{1-\varepsilon}}{\varepsilon}.$$
    Le 😄 Farceur


  • Alors tu y es presque ! Reviens tout simplement à la définition de $\sqrt n a_n \to \frac12$.
  • Merci je vais regarder
    Le 😄 Farceur


  • Ou utilise les $\limsup/\liminf$ si tu connais (ou même toute valeur d'adhérence).
  • Lol comme je suis bête et ça résout ce problème, j'ai honte c'est comme une gifle (je connais les limsup et inf qui existent toujours).
    Merci Simeon
    Le 😄 Farceur


  • En réfléchissant à la propriété que j'ai mentionnée dans mon post précédent, il me semble que (en utilisant la même démonstration que ci-dessus)
    - Si $(u_n)_{n\in\N}$ est une suite décroissante de réels positifs tels que $\displaystyle{S_n=\sum_{k=0}^n u_k}$ est équivalente à $n^a$, où $a\in ]0,1]$, alors $u_n\sim a n^{a-1}$.
    - Si $(u_n)_{n\in\N}$ est une suite croissante de réels positifs tels que $\displaystyle{S_n=\sum_{k=0}^n u_k}$ est équivalente à $n^a$, où $a\in [1,+\infty[$, alors $u_n\sim a n^{a-1}$.
  • Oups, j'avais loupé ton message... Merci!

    En effet, on a l'intuition que si la variation de la suite est trop rapide, il est possible que cela ne marche pas.
  • Dans les cas

    1-$S_n\sim \ln(n)$ ca donne aussi $u_n \sim \frac 1n$ ?
    2-$S_n\sim n\ln(n)$ ca donne aussi $u_n \sim \ln (n)$ ?
    3-$S_n\sim \frac 1 n$ ca donne aussi $u_n \sim \frac 1{n^2}$ ? merci Mrj
    Le 😄 Farceur


  • Pour les deux premiers oui, par contre pour le dernier ça ne va pas marcher (je te laisse réfléchir au pourquoi).
  • la 3 C'est une étourderie - il ne faut pas faire confiance à sa mémoire "confinée" on a plutôt
    $$\sum_{k=n+1}^{+\infty }\dfrac{1}{k^2}\sim \frac 1 n $$
    Le 😄 Farceur


  • En réponse à gebrane:

    Le 2) est une application de http://www.les-mathematiques.net/phorum/read.php?4,1962114,1964704#msg-1964704 avec $\alpha = 1$ et $L = \ln$ (qui est bien à variation lente).

    Il me semble que 1) est faux et que $n \mapsto e^{-\lfloor \ln(n)\rfloor}$ est un contre-exemple (à un facteur près).
    Cependant le cas $\alpha = 0$ du résultat cité donne nécessairement $n u_n = o(\ln(n))$.

    Pour le 3) tu peux a priori adapter aux restes les arguments utilisés.

    Edit : voir http://www.les-mathematiques.net/phorum/read.php?4,1962114,1965356#msg-1965356
Connectez-vous ou Inscrivez-vous pour répondre.